Unsolved Textbook Exercises: Seeking Help and Solutions

  • Thread starter kelvintc
  • Start date
  • Tags
    Textbooks
In summary: OK, so we've solved part a). Now, we need to find the gradient of \vec{T} on the second surface.We can do this by substituting our known \vec{r}\cdot\nabla into the equation for \vec{T}:\vec{T}=x\frac{\partial}{\partial{x}}+y\frac{\partial}{\partial{y}}+z\frac{\partial}{\partial{z}}Now, we need to find the first and last terms on the right-hand side of the equation.The first term is just \vec{T} itself:\vec{T}=x
  • #36
oic... about no.5 it asks me to prove the Stoke's Theorem.
i get [tex] \nabla\times{F}={-x^2}{a}_{z}[/tex]
to find [tex]\int(\nabla\times{F})\cdot{dS}[/tex] , do i need to separate the triangle to 2 pieces, one is x=0 to x=1, y=0 to y=1 and another one is x=1 to x=2, y=1 to y=0 ?
I can't get 7/6 with both ways i did.
But, [tex]\oint{F}\cdot{dl}[/tex] i got 7/6.. curious..
 
Physics news on Phys.org
  • #37
i'm taking electrical engineering...
 
  • #38
1.We agree on the expression on the curl
2. I don't see the need to split up the triangle (I'll get into that)
3. I took a master's in fluid mechanics some time ago..
 
  • #39
My choices for the two triangles:
0<=x<=1,0<=y<=x triangle 1

1<=x<=2, 0<=y<=2-x triangle 2
 
  • #40
yet, i can't get the 7/6. my answer for triangle 1 = -1/4, then triangle 2 = -19/40... sums up can't get 7/6.. anything wrong?

master! oh, it's incredible..
 
Last edited:
  • #41
about (1) since z=0, [tex]a_{r}[/tex] has no [tex]a_{x}[/tex] component , [tex]a_{\phi} = -\sin{\phi}{a}_{x} + \cos{\phi}{a}_{y}[/tex]
[tex]e^{-2}\sin\frac{\phi}{2}*-\sin\phi=-0.102[/tex]
 
  • #42
no.6 i translate all x,y,z into cylindrical coordinates.
[tex]x = r\cos\phi[/tex]
[tex]y = r\sin\phi[/tex]
[tex]z = z[/tex]
[tex]a_{x} = \cos{\phi}{a}_{r} - \sin{\phi}{a}_{\phi}[/tex]
[tex]a_{y} = \sin{\phi}{a}_{r} + \cos{\phi}{a}_{\phi}[/tex]
[tex]a_{x} = a_{z}[/tex]
[tex]dS = rd\phi{d}{r}a_{r}[/tex]
Hence,
[tex]F_{r} = r^2\cos^3\phi + r^2\sin^3\phi[/tex]
integrate i can't get the answer also..
 
Last edited:
  • #43
Hi again!
I got 7/6 in 5); I'll take that first.

1. 7/6 is the number you get by the line integral going around the surface in one direction; -7/6 if you go in the opposite direction

2. We let the normal vector in the surface integral be [tex]\vec{n}=-\vec{a}_{z}[/tex]

3. Hence, we must calculate I:
[tex]I=\int_{0}^{1}\int_{0}^{x}x^{2}dydx+\int_{1}^{2}\int_{0}^{2-x}x^{2}dydx[/tex]

The first integral is: [tex]I_{1}=\frac{1}{4}[/tex]
The second integral is: [tex]I_{2}=\frac{11}{12}[/tex]

This yields 7/6 in total..
 
Last edited:
  • #44
thanks again.. hehe... oh... i really made too much mistakes in calculation.. sad
 
  • #45
No wonder you're stuck by 6)!

As far as I can see, the surface integral (the flux) on the cylindrical shell is 0.

However, if you consider the fluxes through the top and bottom disks (z=0 and z=2) as well, you'll get for the total flux [tex]16\pi[/tex] which seems to agree with the answer..
 
  • #46
Please post some work on 1)
 
  • #47
replace z=0 and z=2 to get [tex](z^2-1)[/tex] ?
then find [tex]\int{rd\phi}{dr} [/tex] ?
by this i get [tex]16\pi[/tex]...
i also get the surface integral in r direction = 0
 
  • #48
(1) [tex]a_{r} = \cos{\phi}{a}_{x} + \sin{\phi}{a}_{y}[/tex]
[tex]a_{\phi} = -\sin{\phi}{a}_{x} + \cos{\phi}{a}_{y}[/tex]
[tex]a_{z} = a_{z}[/tex]
then z=0, so vector in x direction only depends on phi .
[tex]-e^{-2}\sin\frac{\phi}{2}\sin\phi=-0.0586[/tex]
 
Last edited:
  • #49
I'm not sure if your post 47 is a cry for help, or if you got my meaning.
I'll look into 1) later
 
  • #50
i think i got the answer for (6). just not sure whether the method is wrong or not
 
  • #51
Post in detail how you get the answer in 6)
 
  • #52
i got 4 for z=0 to z=2 for (z^2-1) then integrate get 2pie x r since r=2 then 4 times 4pie get 16pie
 
  • #53
Ok, you've got two disks you've got to integrate over in 6)

a) The disk placed at z=2.
At that plane, we have :
[tex]\vec{F}\cdot\vec{n}=(z^{2}-1)|_{(z=2)}=3[/tex]
Or, it's contribution is: [tex]3*\pi(2)^{2}=12\pi[/tex]

b) The disk placed at z=0
At that plane, we have:
[tex]\vec{F}\cdot\vec{n}=(1-z^{2})|_{(z=0)}=1[/tex]
Or, it's contribution is: [tex]\pi(2)^{2}=4\pi[/tex]

Is this how you thought?
 
  • #54
ya... i did it like that
 
  • #55
i had been doing these qs over the past few weeks and there are still some questions remained unsolved.. anyone can help ?
q3, 7, 8, 9, 13, 14, 15
 

Similar threads

  • Introductory Physics Homework Help
Replies
8
Views
1K
  • Science and Math Textbooks
Replies
19
Views
863
  • Science and Math Textbooks
Replies
11
Views
2K
  • Introductory Physics Homework Help
Replies
13
Views
594
  • Science and Math Textbooks
Replies
6
Views
2K
Replies
5
Views
1K
  • Introductory Physics Homework Help
Replies
28
Views
4K
  • STEM Educators and Teaching
Replies
24
Views
2K
  • STEM Academic Advising
Replies
10
Views
2K
  • Introductory Physics Homework Help
Replies
9
Views
2K
Back
Top